Los polos del propagador de Feynman en el espacio de posiciones

Esta pregunta puede estar relacionada con el propagador de Feynman en el espacio de posición a través del parámetro de Schwinger . El propagador de Feynman se define como:

GRAMO F ( X , y ) = límite ϵ 0 1 ( 2 π ) 4 d 4 pags mi i pags ( X y ) pags 2 metro 2 + i ϵ
= { 1 4 π d ( s ) + metro 8 π s H 1 ( 1 ) ( metro s ) s 0 i metro 4 π 2 s k 1 ( metro s ) s < 0

usando ( + , , , ) Convención de firmas de Minkowski.

Si uno quiere aplicar el truco de la rotación de Wick , entonces debe saber la posición de los polos. Es fácil ver que los polos pags 0 de Δ ( pags ) = 1 pags 2 metro 2 + i ϵ son pags 0 = ± ( ω i ϵ ) . Entonces, mi pregunta es ¿cuáles son los polos? X 0 o t de

Δ ( X ) = GRAMO F , ϵ ( X ) = d 4 pags mi i pags X pags 2 metro 2 + i ϵ .

Lo he intentado de la siguiente manera:

Porque

Δ ( pags ) = 1 pags 2 metro 2 + i ϵ = i 0 d α   mi i ( pags 2 metro 2 + i ϵ ) α
De este modo
Δ ( X ) = d 4 pags ( 2 π ) 4 mi i pags X Δ ( pags ) = i 0 d α d 4 pags ( 2 π ) 4   mi i pags X + i ( pags 2 metro 2 + i ϵ ) α = i 0 d α 1 ( 2 π ) 4 [ i π 2 α 2 mi i X 2 4 α i ( metro 2 i ϵ ) α ]
Dejar β = 1 α , entonces obtenemos
1 dieciséis π 2 0 d β   mi i β X 2 4 i ( metro 2 i ϵ ) β
Pero, ¿cómo hacer la última integración y cuáles son los polos? X 0 ?

pd: Este material de Yuri Makeenko (página 8) da una figura para mostrar los polos y las direcciones de rotación de Wick.ingrese la descripción de la imagen aquí

¿Por qué crees que hay diferentes polos? Por cierto: el polo pags 0 = ± ( ω i ϵ ) solo es correcto a primer orden en ϵ .
@Marcel Gracias! Vi en algún material, ver la publicación actualizada por favor.
No estoy seguro de qué se trata esta pregunta. ¿Estás preguntando por los polos del propagador en el espacio de posiciones? ¿Qué te impide calcularlos tú mismo?

Respuestas (2)

Hay una fórmula de integración (ver "Tabla de integrales, series y productos" 7ed, p337 sección 3.324 1ra integral)

0 d β Exp [ A 4 β B β ] = A B k 1 ( A B ) [ R mi A 0 , R mi B > 0 ] .
Si R mi A 0 , R mi B > 0 se viola, la integral será divergente.

En tu caso, A = 4 ( i metro 2 + ϵ ) y B = i X 2 / 4 , asi que R mi A = 4 ϵ > 0 y R mi B = 0 que no satisface la condición convergente. Por lo tanto, para garantizar la convergencia de la integral, debemos tratar B = i X 2 / 4 como el limite B = límite ϵ 0 + i ( X 2 i ϵ ) / 4 . Así tenemos

Δ ( X ) = límite ϵ , ϵ 0 + 1 dieciséis π 2 0 d β Exp [ i β ( X 2 i ϵ ) 4 i ( metro 2 i ϵ ) β ] = límite ϵ , ϵ 0 + 1 4 π 2 metro 2 i ϵ X 2 i ϵ k 1 ( ( metro 2 i ϵ ) ( X 2 i ϵ ) ) = límite ϵ 0 + metro 4 π 2 X 2 i ϵ k 1 ( metro ( X 2 i ϵ ) )
Como resultado, la singularidad del propagador está en X 2 i ϵ = t 2 X 2 i ϵ = 0 , es decir t = ± ( | X | + i ϵ ) .

En realidad, la condición convergente de la integral restringe el régimen analítico de Δ ( X ) :

0 < R mi ( i X 2 ) = R mi ( i t 2 ) = yo metro ( t 2 )
es decir
( 2 norte 1 ) π argumento ( t 2 ) = 2 argumento ( t ) 2 norte π ( norte 1 2 ) π argumento ( t ) norte π
Por lo tanto, Δ ( X ) solo se puede continuar analíticamente hasta el segundo y el cuarto cuadrantes en el plano complejo de t . En conclusión, la rotación de la mecha en t el plano debe ser en el sentido de las agujas del reloj.

¡Muchas gracias! Pero por qué R mi ( i X 2 ) = R mi ( i t 2 ) ? En general, R mi ( i X 2 ) = R mi ( i t 2 i | X | 2 ) .
Lo sé, es porque t = ± ( | X | + i ϵ ) y i | X | 2 es totalmente imaginario.
@EdenHarder Sí, porque solo extendemos t al plano complejo, pero X siempre es real

Los polos descansan sobre el cono de luz, es decir GRAMO ( X , y ) por X y . Para ver esto, intente calcular la integral a través del cálculo de residuos. Primero, realiza una transformación de Lorentz de la variable de integración, de modo que a x solo le quede una entrada (esta es la entrada temporal para x temporal o una espacial para espacial). Ahora para s = X 2 = 0 no hay posibilidad de un factor de amortiguamiento exponencial independientemente de dónde cierre el contorno y la integral divergirá.

Para ver que esas son las únicas singularidades, observe el lado derecho de su ecuación: tiene la función delta que diverge para s=0 y las funciones de Hankel/Bessel que, combinadas con su prefactor, ambas divergen como 1 / s y no tienen otras singularidades.

¡Gracias! ¿Podrías mostrar tu idea mediante fórmulas?
Debe usar muchas representaciones integrales de las funciones de Bessel para calcular explícitamente el propagador. ¿Por qué no echa un vistazo al cálculo aquí , que puede ser exactamente lo que está buscando? Los polos se encuentran en X ² i ϵ = 0, entonces t 1 / 2 = ± ( | X | + i ϵ ) .
Sí, muestran los detalles del cálculo. Pero quiero que alguien me enseñe cómo terminar el último paso en el cálculo que se muestra en la publicación, es decir,
1 dieciséis π 2 0 d β   mi i β X 2 4 i ( metro 2 i ϵ ) β
¡Muchas gracias!